If yam is added fifth, which one of the following could be true?

zia305 on May 23, 2020

Explanation

Can someone please explain the right answer?

Reply
Create a free account to read and take part in forum discussions.

Already have an account? log in

Skylar on May 25, 2020

@zia305, happy to help!

We are asked what could be true if Y is fifth.

Let's start by looking at our rules about what happens when Y is in. The contrapositive of Rule #2 says that if Y is in, J is out. Rule #3 says that Y and P cannot both be in. So, we know that J and P must both be out. This means that there are exactly 5 ingredients in.

__ __ __ __ Y | J P
1 2 3 4 5 OUT

The remaining ingredients that must be in, as confirmed by Rule #4, are G, K, O, and T. Rule #5 further specifies that O precedes G which precedes both K and T. So, we know that O must be first and G must be second. Remember, we don't know the order of K and T. This gives us a final diagram as follows:

O G (K/T) (T/K) Y | J P
1 2 3 4 5 OUT

Using this diagram, we see that the only answer choice that could be true when Y is fifth is (C), that kale is third.

Does that make sense? Please let us know if you have any other questions!